Use non-breaking space (~) between 'Figure' and the figure number.
[course.git] / latex / problems / Serway_and_Jewett_8 / problem09.56.tex
1 \begin{problem*}{9.56}
2 Figure~P9.56 shows three points in the operation of the ballistic
3 pendulum discussed in Example 9.6 (and shown in Fig.~9.9b).  The
4 projectile approaches the pendulum in Figure~P9.56a.  Figure~P9.56b
5 shows the situation just after the projectile is captured in the
6 pendulum.  In Figure~P9.56c, the pendulum arm has swung upward and
7 come to rest at a height $h$ above its initial position.  \Part{a}
8 Prove that the ratio of kinetic energy of the projectile-pendulum
9 system immediately after the collision to the kenetic energy
10 immediately before is $m_1/(m_1+m_2)$.  \Part{b} What is the ratio of
11 the momentum of the system immediately after the collision to the
12 momentum immediately before?  \Part{c} A student believes that such a
13 large decrease in mechanical energy must be accompanied by at least a
14 small decrease in momentum.  How would you convince this student of
15 the truth?
16 \end{problem*}
17
18 \begin{solution}
19 \end{solution}